Menu Close

Author: Tinku Tara

Question-137317

Question Number 137317 by physicstutes last updated on 01/Apr/21 Commented by physicstutes last updated on 01/Apr/21 $$\mathrm{The}\:\mathrm{figure}\:\mathrm{above}\:\mathrm{shows}\:\mathrm{a}\:\mathrm{composite}\:\mathrm{bar} \\ $$$$\mathrm{made}\:\mathrm{of}\:\mathrm{two}\:\mathrm{materials}\:{X}\:\mathrm{and}\:{Y}.\:\mathrm{The}\:\mathrm{end} \\ $$$$\mathrm{of}\:{X}\:\mathrm{is}\:\mathrm{maintained}\:\mathrm{at}\:\mathrm{100}°\mathrm{C}\:\mathrm{and}\:\mathrm{it}\:\mathrm{has} \\ $$$$\mathrm{length}\:\mathrm{of}\:\mathrm{25}\:\mathrm{cm}\:\mathrm{while}\:\mathrm{that}\:\mathrm{of}\:{Y}\:\mathrm{is}\:\mathrm{maintained} \\ $$$$\mathrm{at}\:\mathrm{0}°\mathrm{C}\:\mathrm{with}\:\mathrm{lenght}\:\mathrm{75}\:\mathrm{cm}.\:\mathrm{Find}\:\mathrm{the}…

Show-that-x-R-the-sequence-f-n-0-defined-by-f-0-cosx-f-1-sin-cosx-and-f-n-sin-cos-f-n-2-if-n-3-is-odd-cos-sin-f-n-2-if-n-2-is-even-converges-to-a-limit-l-0-6-0-7

Question Number 6247 by Yozzii last updated on 20/Jun/16 $${Show}\:{that},\:\forall{x}\in\mathbb{R},\:{the}\:{sequence}\:\left\{{f}\left({n}\right)\right\}_{\mathrm{0}} ^{\infty} \\ $$$${defined}\:{by}\:{f}\left(\mathrm{0}\right)={cosx},\:{f}\left(\mathrm{1}\right)={sin}\left({cosx}\right)\:{and} \\ $$$${f}\left({n}\right)=\begin{cases}{{sin}\left({cos}\left({f}\left({n}−\mathrm{2}\right)\right)\right)\:\:{if}\:{n}\geqslant\mathrm{3}\:{is}\:{odd}}\\{{cos}\left({sin}\left({f}\left({n}−\mathrm{2}\right)\right)\right)\:\:{if}\:{n}\geqslant\mathrm{2}\:{is}\:{even},}\end{cases} \\ $$$${converges}\:{to}\:{a}\:{limit}\:{l}\in\left(\mathrm{0}.\mathrm{6},\mathrm{0}.\mathrm{7}\right). \\ $$$${If}\:{you}\:{can},\:{determine}\:{the}\:{exact}\:{value} \\ $$$${of}\:{l}. \\ $$ Commented by…

An-alternating-current-after-passing-through-rectifire-has-the-form-i-I-0-sinx-for-0-x-pi-0-for-pi-x-2pi-where-I-0-is-the-maximum-current-and-period-is-

Question Number 137316 by BHOOPENDRA last updated on 01/Apr/21 $${An}\:{alternating}\:{current}\:{after}\:{passing}\: \\ $$$$\:{through}\:{rectifire}\:{has}\:{the} \\ $$$${form}\: \\ $$$$\:\:\:\:\:\:\:\:\:\:\:\:{i}={I}_{\mathrm{0}} {sinx}\:\:\:\:\:\:{for}\:\mathrm{0}\leqslant{x}\leqslant\pi \\ $$$$\:\:\:\:\:\:\:\:\:\:\:\:\:\:\:\:=\mathrm{0}\:\:\:\:\:\:\:\:\:\:\:\:\:{for}\:\pi\leqslant{x}\leqslant\mathrm{2}\pi \\ $$$${where}\:{I}_{\mathrm{0}} \:{is}\:{the}\:{maximum}\:{current}\: \\ $$$${and}\:{period}\:{is}\:\mathrm{2}\pi.{express}\:{i}\:{is}\:{a}\: \\…

Need-help-solving-0-x-x-dx-My-current-working-x-x-e-xlnx-e-xlnx-1-xln-x-x-2-ln-x-2-2-x-3-ln-x-3-3-n-0-1-n-x-n-ln-n-x-n-0-x-

Question Number 6246 by FilupSmith last updated on 20/Jun/16 $$\mathrm{Need}\:\mathrm{help}\:\mathrm{solving}\:\int_{\mathrm{0}} ^{\:\infty} {x}^{−{x}} {dx} \\ $$$$\mathrm{My}\:\mathrm{current}\:\mathrm{working}: \\ $$$${x}^{−{x}} ={e}^{−{x}\mathrm{ln}{x}} \\ $$$${e}^{−{x}\mathrm{ln}{x}} =\mathrm{1}−{x}\mathrm{ln}\left({x}\right)+\frac{{x}^{\mathrm{2}} \mathrm{ln}\left({x}\right)^{\mathrm{2}} }{\mathrm{2}!}−\frac{{x}^{\mathrm{3}} \mathrm{ln}\left({x}\right)^{\mathrm{3}} }{\mathrm{3}!}+……

Question-71776

Question Number 71776 by TawaTawa last updated on 19/Oct/19 Answered by tw000001 last updated on 22/Oct/19 $$\mathrm{Use}\:\mathrm{Harmonic}\:\mathrm{series}\:\mathrm{to}\:\mathrm{solve}. \\ $$$${A}=\mathrm{1}−\frac{\mathrm{1}}{\mathrm{2}}+\frac{\mathrm{1}}{\mathrm{3}}−\frac{\mathrm{1}}{\mathrm{4}}+…+\frac{\mathrm{1}}{\mathrm{2015}}−\frac{\mathrm{1}}{\mathrm{2016}} \\ $$$$=\left(\mathrm{1}+\frac{\mathrm{1}}{\mathrm{2}}+\frac{\mathrm{1}}{\mathrm{3}}+…+\frac{\mathrm{1}}{\mathrm{2016}}\right)−\mathrm{2}\left(\frac{\mathrm{1}}{\mathrm{2}}+\frac{\mathrm{1}}{\mathrm{4}}+\frac{\mathrm{1}}{\mathrm{6}}+…+\frac{\mathrm{1}}{\mathrm{2016}}\right) \\ $$$$=\left(\mathrm{1}+\frac{\mathrm{1}}{\mathrm{2}}+\frac{\mathrm{1}}{\mathrm{3}}+…+\frac{\mathrm{1}}{\mathrm{2016}}\right)−\left(\mathrm{1}+\frac{\mathrm{1}}{\mathrm{2}}+\frac{\mathrm{1}}{\mathrm{3}}+…+\frac{\mathrm{1}}{\mathrm{1008}}\right) \\ $$$$={H}_{\mathrm{2016}}…

lim-n-n-3-n-n-n-3-n-

Question Number 71777 by Henri Boucatchou last updated on 19/Oct/19 $$\:\:\underset{\boldsymbol{{n}}\rightarrow\infty} {\boldsymbol{{lim}}}\:\left(\frac{\boldsymbol{{n}}!\:+\:\mathrm{3}^{\boldsymbol{{n}}} }{\boldsymbol{{n}}^{\boldsymbol{{n}}} \:+\:\mathrm{3}^{\boldsymbol{{n}}} }\right)\:=\:? \\ $$ Commented by mathmax by abdo last updated on…